Which of the choices is NOT a good example of a line of best fit?

Which Of The Choices Is NOT A Good Example Of A Line Of Best Fit?

Answers

Answer 1

Answer:

B

Step-by-step explanation:

B because the line doesn't go through any of the data points.

Answer 2
The letter B is right

Related Questions

what is the distance between (1,4) and (4,0) ?

Answers

Answer:

5 units

Step-by-step explanation:

To calculate the distance (d) use the distance formula

d = √ (x₂ - x₁ )² + (y₂ - y₁ )²

with (x₁, y₁ ) = (1, 4) and (x₂, y₂ ) = (4, 0)

d = [tex]\sqrt{(4-1)^2+(0-4)^2}[/tex]

   = [tex]\sqrt{3^2+(-4)^2}[/tex]

   = [tex]\sqrt{9+16}[/tex] = [tex]\sqrt{25}[/tex] = 5

14. Solve -4x2 - 7x = -5.

Answers

Answer:

x=−7/8±129/8

Step-by-step explanation:

Assuming that is a -4x^2, i'll solve it. So, -4x^2 - 7x = -5. This is a quadratic, so move all the numbers to one side (and variables). So, -4x^2 - 7x + 5 = 0. Divide everything by -1, to make the coefficient of the x squared positive. This leaves: 4x^2 + 7x -5 = 0. Now, factoring attempts: (2x-1)(2x+5). This does not work, sadly, so we must find other methods. Using the quadratic formula would be easier than factoring, so use the quadratic formula. This gives us the answers of -7 plus or minus sqrt(129) all over 8. The quadratic formula comes in handy!

suppose you want to convert kilometers to miles. you use the conversion of 1 mile = 1.6 kilometers. when using this conversion, which unit should be in the denominator?​

Answers

Answer:

The unit that should be in the denominator is kilometers

Step-by-step explanation:

we know that

[tex]1\ mile=1.6\ kilometers[/tex]

To convert x km to mi

[tex]x\ km*(\frac{1}{1.6})\frac{mi}{km}=\frac{x}{1.6}\ mi[/tex]

therefore

The unit that should be in the denominator is kilometers

1 and 3/4 + 2 and 3/8

Answers

Hello There!

1[tex]\frac{3}{4}[/tex] + 2[tex]\frac{3}{8}[/tex] = 4[tex]\frac{1}{8}[/tex]

First, when we are trying to find the sum of a mixed number, I always add the natural numbers first meaning that the numbers before the fraction so I would add 1 and 2 together so we get a sum of 3 and now we are left with just a plain fraction.

Next, I find the least common denominator which is the smallest number that can be a common denominator for a set of fractions. Our lowest common denominator is 8 because [tex]\frac{3}{4}[/tex] = [tex]\frac{6}{8}[/tex].

Then, we add our fractions with the denominator of 8 together and get a sum of 9/8 which we can turn into a mixed number because the numerator is bigger than our denominator.

Our mixed number turns into 1 and 1/8 and we add 4 to it because that was the sum of our natural numbers and get a sum of 4 and 1/8

ANSWER 4 1/8

What is the measure of A

Answers

Answer:

A. 60°

Step-by-step explanation:

From the diagram, in triangle ABC,

AB=BC=CA=15 units.

This means triangle ABC is an equilateral triangle.

All angles in equilateral triangle are congruent. The sum of all interior angles in triangle is always 180°, so one angle of equilateral triangle is equal to 60°. Thus,

∠A=∠B=∠C=60°

What is the volume of a rectangle Kay prism that is 16 meters by 25 meters by 37 meters? PLZ HELP QUICK

Answers

To find the volume multiply the three dimensions:

16 x 25 x 37 = 14,800 cubic meters.

Can anyone answer this ?

Answers

For this case we have that by definition, the Pythagorean theorem states that:

[tex]c = \sqrt {a ^ 2 + b ^ 2}[/tex]

Where:

c: It is the hypotenuse of the triangle

a, b: They are the legs of the triangle

Then, we verify if the theorem for the given triangles is fulfilled:

Triangle 1:

[tex]\sqrt {13} = \sqrt {2 ^ 2 + 3 ^ 2}\\\sqrt {13} = \sqrt {4 + 9}\\\sqrt {13} = \sqrt {13}[/tex]

It is fulfilled!

Triangle 2:

[tex]25 = \sqrt {2 ^ 2 + (3 \sqrt {2}) ^ 2}\\25 = \sqrt {4+ (9 * 2)}\\25 = \sqrt {22}[/tex]

It is not fulfilled!

Triangle 3:

[tex]43 = \sqrt {2 ^ 2 + (3 \sqrt {3}) ^ 2}\\43 = \sqrt {4+ (9 * 3)}\\43 = \sqrt {4+ (9 * 3)}\\43 = \sqrt {31}[/tex]

It is not fulfilled!

ANswer:

Triangle A

which of the following is equivalent to the expression i^88

Answers

Answer:

i^88 =  1

Step-by-step explanation:

i^88 = i^ 4*22 = 1    { i^4k = 1 ; i^4k+1 =i ; i^4k+2 = (-1); i^4k+3 = (-i)  }

Final answer:

The equivalent expression for i^88 is 1 as the powers of i cycle every 4. Hence (i^4)^22 which is same as 1^22 ends up being 1.

Explanation:

In complex numbers, i is the imaginary unit with the property i^2 = -1. The powers of i repeat in a cycle: i^1=i, i^2=-1, i^3=-i, and i^4=1. To find the equivalent expression for i^88, we will the fact that i^4=1 and i^88 would be equivalent to (i^4)^22, because 4*22 = 88.

Therefore, (i^4)^22 = 1^22. The equivalent expression for i^88 is 1.

Learn more about Imaginary Numbers here:

https://brainly.com/question/13174285

#SPJ11


Which of the following is the result of using the remainder theorem to find F(-2)
for the polynomial function F(x) = -2x3 + x2 + 4x-3?
A. 9
B. -11
C.3
D. -23

Answers

Answer:

A. 9

Step-by-step explanation:

F(-2) = 9

We are given the polynomial function;

F(x) = -2x3 + x2 + 4x-3

In order to determine F(-2) using the remainder theorem, we plug in -2 in place of x in the equation and simplify;

F(-2) = -2(-2)^3 + (-2)^2 + 4(-2) - 3

F(-2) = 9

Answer:

A

Step-by-step explanation:

Evaluating F(- 2) gives the remainder on dividing the polynomial by (x + 2)

F(- 2) = - 2(- 2)³ + (- 2)² + 4(- 2) - 3 = 16 + 4 - 8 - 3 = 9 ← remainder

write y+1=-2x-3 in standard form​

Answers

Answer:

2y=-5x

Step-by-step explanation:

First you mark your terms, which means to basically put the equation in order.

Then you add, for instance, first you'll add y and 1, you'll have 2y because y means one. Then, you'll have -2 minus 3 and you'll get -5 then you carry on the variables to the solutions.

Answer:

2y=-5x

hope this helps :3

Question 1
The number of laptop computers sold each month for one year was
recorded by an electronics store. The results were 14, 15, 15, 30, 29, 5, 9, 15,
21, 21, 26, and 15. Calculate the median number of laptop computers sold
per month.​

Answers

Final answer:

The median number of laptop computers sold per month is 15, calculated by arranging the sales data in ascending order and averaging the two middle values in the even-numbered dataset.

Explanation:

The median of laptop computers sold per month can be calculated by first arranging the given numbers in ascending order and then finding the middle value. If there is an even number of observations, the median is the average of the two middle numbers.

Arrange the sales numbers in ascending order: 5, 9, 14, 15, 15, 15, 15, 21, 21, 26, 29, 30.Since there are 12 months, we have an even number of observations, so we take the average of the 6th and 7th values which are both 15.The median number of laptop computers sold per month is 15.

What is the completely factored form of d4 − 81?
(d + 3)(d − 3)(d + 3)(d − 3)
(d2 + 9)(d + 3)(d − 3)
(d2 + 9)(d − 3)(d − 3)
(d2 + 9)(d2 − 9)

Answers

For this case we must factor the following expression:

[tex]d ^ 4-81[/tex]

Rewriting the expression:

[tex](d ^ 2) ^ 2-9 ^ 2[/tex]

We factor using the formula of the square difference:

[tex]a ^ 2-b ^ 2 = (a + b) (a-b)[/tex]

Where:

[tex]a = d ^ 2\\b = 9[/tex]

So:

[tex](d ^ 2 + 9) (d ^ 2-9)[/tex]

From the second term we have:

[tex]d ^ 2-3 ^ 2 = (d-3) (d + 3)[/tex]

Finally, the factored expression is:

[tex](d ^ 2 + 9) (d-3) (d + 3)[/tex]

Answer:

[tex](d ^ 2 + 9) (d-3) (d + 3)[/tex]

Answer:

The complete factorization of the term:

              [tex]d^4-81[/tex] is:

        [tex](d-3)(d+3)(d^2+9)[/tex]

Step-by-step explanation:

To factor a term means to express is as a product of distinct factors i.e. multiples.

We are asked to factor the algebraic expression which is given by:

[tex]d^4-81[/tex]

We could write this expression as:

[tex](d^2)^2-(3^2)^2=(d^2)^2-(9)^2[/tex]

We know that:

[tex]a^2-b^2=(a-b)(a+b)[/tex]

i.e.

[tex]d^4-81=(d^2-9)(d^2+9)\\\\i.e.\\\\d^4-81=(d^2-3^2)(d^2+9)\\\\i.e.\\\\d^4-81=(d-3)(d+3)(d^2+9)[/tex]

How to round 56 to the nearest ten

Answers

Answer:

The answer would be 60

Step-by-step explanation:

What you do is if the number if 5 or less (ex 55) you would round down to 50. However it is 56 so you would round up to the next highest tenth, 60.

Hope this helps! Have a great day!

[tex]\text{Hey there}[/tex]

[tex]\text{If you come across: 5, 6, 7 , 8 , \& 9 you're going}\uparrow\text{(up)}[/tex]

[tex]\text{If you come across 1 , 2 , 3 , \& 4 you're going}\downarrow\text{(down)}[/tex]

[tex]\text{In this equation we have a SIX (6) at the end of the equation so we're going UP!}[/tex]

[tex]\boxed{\boxed{\bf{Ansewr: 60}}}}\checkmark[/tex]

[tex]\text{Good luck on your assignment and enjoy your day!}[/tex]

~[tex]\frak{LoveYourselfFirst:)}[/tex]

Which is a solution to the equation?
(х-2)(х + 5) = 18?

Answers

[tex]\bf (x-2)(x+5)=18\implies \stackrel{\mathbb{F~O~I~L}}{x^2+3x-10}=18\implies x^2+3x-28=0 \\\\\\ (x-4)(x+7)=0\implies x= \begin{cases} 4\\ -7 \end{cases}[/tex]

4x(3x-7)-19x^2 simplify the expression below

Answers

Answer:

opening the bracket, the expression becomes

12x^2-28-19x^2

collect like terms

12x2-19x^2-28

-7x^2-28

-7(x^2+4)

find the length of AB leave your answer in terms of pi help please ​

Answers

[tex]\bf \textit{arc's length}\\\\ s=\cfrac{\pi \theta r}{180}~~ \begin{cases} r=radius\\ \theta =\textit{angle in}\\ \qquad \textit{degrees}\\ \cline{1-1} r=6\\ \theta =30 \end{cases}\implies s=\cfrac{\pi (30)(6)}{180}\implies s=\pi[/tex]

The values in the table represent an exponential function. What is the common ratio of the associated geometric sequence?

Answers

Answer:

D. 3

Step-by-step explanation:

[tex]a_1,\ a_2,\ a_3,\ ...,\ a_n-\text{geometric series}\\\\r=\dfrac{a_2}{a_1}=\dfrac{a_3}{a_2}=\dfrac{a_4}{a_3}=...=\dfrac{a_n}{a_{n-1}}-\text{common ratio}\\\\\text{From the table we have:}\\\\a_1=7,\ a_2=21,\ a_3=63,\ a_4=189,\ a_5=567\\\\\text{Check the common ratio:}\\\\\dfrac{21}{7}=3\\\\\dfrac{63}{21}=3\\\\\dfrac{189}{63}=3\\\\\dfrac{567}{189}=3\\\\\bold{CORRECT}[/tex]

Interest rate is 4.25%, the time is 3 1/4 years, simple interest is $330. What is the principal?

Answers

Answer:

$2389.14

Step-by-step explanation:

The equation is I = PRT

330= (0.0425)(3.25)P

P=2389.14

Answer:

$2389.14.

Step-by-step explanation:

Use the formula

I =  PRT/100 where I = interest , P  is the principle, t = time and R = the rate.

330 = P * 4.25 * 3.25 / 100

33000 = P * 13.8125

P = 33,000 / 13.8125

P = $2389.14.

Hamid ha gained weight, he now weighs 88kg which is 10% higher than the normal, what i Hamid normal weight?

Answers

Answer:

80 kg is Hamid's normal weight

Step-by-step explanation:

The following equation will help you to find the answer:

(1.10)x = 88

Answer:79.2kg

Step-by-step explanation:

10%of 88kg is 8.8. Subtract 8.8 from 88 and the answer is 79.2.

If f(x) = 3x + 10 and g(x) = 2x– 4, find (f+ g)(x).

Answers

Answer:

5x+6

Step-by-step explanation:

f(x) = 3x + 10

g(x) = 2x– 4

(f+ g)(x) = 3x+10 + 2x-4

Combine like terms

           = 5x+6

Answer:

5x + 6

Step-by-step explanation:

 (f+ g)(x) = 3x + 10 +  2x– 4

               = 5x + 6

The first two steps in determining the solution set of the system of equations, y = x2 - 2x - 3 and y = -x +3, algebraically are
shown in the table.
Step
Equation
Step 1 XP-2x-3--x+3
Step 2
0=x2-X-6
Which represents the solution(s) of this system of equations?
(3.0) and (-2,5)
(-6, 9) and (1, 2)
(-3,6) and (2, 1)
(6.-3) and (-1.4)

Answers

Answer:

(3,0) and (-2,5)

Step-by-step explanation:

The solutions of the equations are A ( 3 , 0 ) and B ( -2 , 5 ) and the graph is plotted

What is an Equation?

Equations are mathematical statements with two algebraic expressions flanking the equals (=) sign on either side.

It demonstrates the equality of the relationship between the expressions printed on the left and right sides.

Coefficients, variables, operators, constants, terms, expressions, and the equal to sign are some of the components of an equation. The "=" sign and terms on both sides must always be present when writing an equation.

Given data ,

Let the equation be represented as C

Now , the value of C is

Substituting the values in the equation , we get

y = x² - 2x - 3   be equation (1)

y = -x + 3   be equation (2)

On simplifying , we get

-x + 3 = x² - 2x - 3

Adding x on both sides , we get

x² - 3 - x = 3

Subtracting 3 on both sides , we get

x² - x - 6 = 0

On factorizing , we get

( x - 3 ) ( x + 2 ) = 0

So , the two values of x are 3 and -2

Hence , the solutions of equations are A ( 3 , 0 ) and B ( -2 , 5 )

To learn more about equations click :

https://brainly.com/question/19297665

#SPJ7

the sum of three consecutive natural numbers is 156 find the number which is the multiple of 13 out of these numbers​

Answers

Answer:

52 is the multiple of 13

Step-by-step explanation:

3x+3=156

3x=153

x=52

Answer:

52 is the multiple of 13 out of 51 , 52, 53 numbers​.

Step-by-step explanation:

Given:  Sum of three consecutive integers 156

To find: Three consecutive integers .

Solution: We have given that

Let first consecutive number  x ,

Second consecutive number=   x+1

Third number =  x+2

According to question :

Sum of three consecutive number

x + x+1 +x+2 = 156 .

Combine like term

3x+3 = 156

On subtracting by 3 both side

3x + 3 -3 = 156 - 3

3x = 153

On dividing by 3

x = 51.

X+1 = 51+1

x+1 = 52.

x +3 = 51+2 = 53.

We can see second number 52 is multiple of 13.

Therefore, 52 is the multiple of 13 out of 51 , 52, 53 numbers​.

6 x j = 42 ??????? Help

Answers

Answer:

j = 7

Step-by-step explanation:

flip the equation around: instead of using multiplication you use division to find out what j is.

1st step: 42 divided by 6 is 7

2nd step: (check your answer): 7 times 6 does equal 42, therefore j = 7 is correct.

Answer:

[tex]\huge \boxed{J=7}[/tex]

Step-by-step explanation:

Switch sides.

[tex]\displaystyle6j=42[/tex]

Divide by 6 from both sides.

[tex]\displaystyle \frac{6j}{6}=\frac{42}{6}[/tex]

Simplify, to find the answer.

[tex]\displaystyle 42\div6=7[/tex]

[tex]\huge \boxed{j=7}[/tex], which is our answer.

What is the slope of the line shown in the graph?
A) 3/2
B) 2/3
C) -3/4
D) -2/3

Answers

Choose two points

(4,1) and (-3 , 5)

rise/run = 4/6

Simplify - 2/3

Answer = B) 2/3

Hope this helps!!

we can simply get the slope by using two points off the line, hmmmm say the line passes through (0,3) and (-3,5)

[tex]\bf (\stackrel{x_1}{0}~,~\stackrel{y_1}{3})\qquad (\stackrel{x_2}{-3}~,~\stackrel{y_2}{5}) \\\\\\ slope = m\implies \cfrac{\stackrel{rise}{ y_2- y_1}}{\stackrel{run}{ x_2- x_1}}\implies \cfrac{5-3}{-3-0}\implies \cfrac{2}{-3}\implies -\cfrac{2}{3}[/tex]

A change machine can accept $1, $5, $10, and $20 bills and returns quarters. What is the domain and range of this situation?

Answers

Answer:

Domain {1,5,10,20}

Ranger {4,20,40,80}

Step-by-step explanation:

$1=4 quarters

$5=20 quarters

$10=40 quarters

$20=80 quarters

Domain {1,5,10,20}

Ranger {4,20,40,80}

What are the x- and y-intercepts for the equation "2x + 3y = 6"?
(2,0) & (3,0)
(0, 2) & (3, 0)
(0,3) & (2,0)
(0,3) & (0,2)​

Answers

Answer:

(0,2) & (3,0)

Step-by-step explanation:

Given the equation [tex]2x+3y=6[/tex]:

Step 1:

To find the y-intercept, you want to set the x value to zero. This will allow you to solve for y, and find where the equation intercepts why on the 0 line:

[tex]2(0)+3y=6 \\ 0+3y=6\\ 3y=6\\ y=\frac{6}{3} \\  y = 2[/tex]

So, at x=0, y=2. or (0,2)

Step 2:

Set y = 0 and solve for x:

[tex]2x+3(0)=6\\ 2x+0=6\\ 2x=6\\ x=\frac{6}{2}\\  x=3[/tex]

so at y=0, x=3. which is the same as saying: when x=3, y=0, or (3,0)

2x+3y=6
The answer is
(0,2) & (3,0)
You have to put 0 in the x then solve for y
After u get y replace the number in y and solve for x

At a point on the ground 60 Ft
from the base of a tree, the distance
to the top of the tree is 4 ft more
than 2 times the height of the tree.
Find the height of the tree in ft.​

Answers

The height of the tree is 32 feet.

Step-by-step explanation:

Let the height of the tree be x.

Connect the top of the tree, the point on the ground, and the bottom of the tree to form a right triangle.

Use Pythagorean theorem to solve for x.

(4+2x)^2 = x^2 + 60^2

x = 32

Final answer:

The height of the tree is 4 feet.

Explanation:

To find the height of the tree, we can set up a right triangle with the tree height as one leg, the distance from the base of the tree as the other leg, and the distance to the top of the tree as the hypotenuse. Let's call the tree height x. From the given information, we can write the equation:

x = 2x + 4

Simplifying this equation, we get:

x = 4

Therefore, the height of the tree is 4 feet.

Learn more about Geometry here:

https://brainly.com/question/31408211

#SPJ3

Solve: x + 3 = –x + 7

Answers

Answer:

x = 2

Step-by-step explanation:

x+3 = -x + 7

x = -x + 4

(2) = -(2) + 4

2 = 2

Answer:

The correct answer is x = 2.

Step-by-step explanation:

To solve this equation, we must get all of the variables (x's) to one side of the equation and get all of the constant terms (numbers only) to the other side.

Starting with this equation, we are going to add x to both sides of the equation.  This will cancel out the -x on the right side of the equation, thereby moving all of the variable x's to the left side of the equation.

x + x + 3 = -x + x + 7

If we simplify by combining like terms, or adding the x's together, we get:

2x + 3 = 7

Next, we should subtract 3 from both sides of the equation in order to move all of the constant terms to the right side of the equation.

2x + 3 -3 = 7 - 3

If we simplify, we get:

2x = 4

Finally, we should divide both sides by 2 in order to get the variable x alone on the left side of the equation, solving it.

x = 2

Therefore, your answer is x = 2.

Hope this helps!

How do you do this question down below?

Answers

Answer:

y = 3m - 6

Step-by-step explanation:

y = mx + b

b is the point where the line cuts the y axis.

That happens at (0,-6)

So far what you have on this equation is

y = mx - 6

You could use the point that cuts the x axis to find m.

y = 0

x = 2

0 = m*2 - 6                  Add 6 to both sides

6 = m*2 - 6 + 6

6 = 2*m                        Divide by 2

6/2 = 2m/2                   Do the division

3 = m

Answer

y = 3m - 6

Help me out again Please

Answers

1= 25
2= 50
3= 25x3 = 75

Or you can do 1/25 = 3/x
X= 75

100/25 = 4


25x 12 = 300
If this is confusing. Write the answers on the following


First box is 75
Second box is 4
Third box is 300

Hope this helps!
Other Questions
The value of k so that the sequence k-1, k+3, 3k-1 forms an arithmetic progression is? Where is a thesis statement revealed? In an essays introduction In an essays argument In an essays body In an essays conclusion The fact that ethnicity is defined as a socially constructed categorization tells the reader that ethnicity? Which of the following is a "characteristic" of the sympathetic divisiona. Urinationb. Excitementc. Digestiond. Defecatione. Salivation Which kind of excersie helps your heart work more efficiently When should you begin preparing for an interview?A. At the beginning stages of your job searchB. After you complete your applicationC. After youve applied to several jobs When did the first economies begin to develop?When all the hunter-gatherers diedWhen money was inventedWhen people began to grow crops and domesticate animalsWhen the first cities were built What is a negative influence on community health?A: National health debt B: Citizens who drive unsafely C: Unreliable public transportation D: Stores that sell tobacco to minors How did the second great awakening influence american society? Clara invit a sus amigas a comer en un restaurante que est cerca de la universidad. Decidieron compartir varios platos. Pidieron camarones fritos, queso y verduras con salsa de yogur como entremeses. Tambin pidieron una ensalada con aceite y vinagre. Bebieron agua mineral. Despus el camarero les sirvi chuletas de cerdo con papas al horno (baked potatoes) y pimientos rellenos de langosta (lobster stuffed peppers). Como la comida y el servicio (service) fueron excelentes, Clara y sus amigas le dejaron una muy buena propina al camarero. 1. Dnde est el restaurante? 2. Las amigas decidieron... 3. Qu tipo de ensalada pidieron? 4. Qu bebieron? 5. Con qu les sirvieron las chuletas de cerdo? Which term describes combining two incomplete protein sources to make up for amino acids that are lacking in the other food? please help1. f(x)=3(2)^x2. f(x)=2(3)^x3. f(x)=3(1/2)^x4. f(x)=2(1/3)^x PLEASE HELP!!!!!!!!!!!!!!!!! Type the correct answer in each box. Spell all words correctly, and use numerals instead of words for numbers. The formula for eccentricity, e, of an orbit is given below, where a is the length of the semi-major axis and b is the length of the major axis. Which of the following has the most mass? A hot air balloon 20 bowling balls A canoe 30 ounces of lead PLS HELP OMG-what is the best estimate of a 20% tip if you ordered a meal that cost $29.35? What are the names of the two Aztec calendars? What do they keep track of? Emmanuel is caring for Mr. Natik, a 61-year-old Caucasian male who has just undergone cataract surgery in his right eye. Emmanuel teaches Mr. Natik safety precautions, including preventing the risk of burns while cooking. A self-inflicted burn is referred to as a: ______ ________ _________ The middle east is especially rich in deposits of Please answer this multiple choice question CORRECTLY for 30 points and brainliest!! CategorasMultiple choice Activity InstructionsSelect the item that does not belong. May 21 12:00 PM 4 attempts remaining Grade settings External references152-157 Vocabulary list Questions 1. el ascensor el viajero el empleado el husped 2. montar a caballo pescar acampar jugar a las cartas 3. el pasaje la playa el mar la tabla de windsurf 4. ir de vacaciones ir de compras ir en avin hacer un viaje 5. el aeropuerto la estacin de autobuses la estacin de tren la habitacin